Stay ahead of learning milestones! Enroll in a class over the summer!

G
Topic
First Poster
Last Poster
Concurrency
Dadgarnia   27
N 2 minutes ago by zuat.e
Source: Iranian TST 2020, second exam day 2, problem 4
Let $ABC$ be an isosceles triangle ($AB=AC$) with incenter $I$. Circle $\omega$ passes through $C$ and $I$ and is tangent to $AI$. $\omega$ intersects $AC$ and circumcircle of $ABC$ at $Q$ and $D$, respectively. Let $M$ be the midpoint of $AB$ and $N$ be the midpoint of $CQ$. Prove that $AD$, $MN$ and $BC$ are concurrent.

Proposed by Alireza Dadgarnia
27 replies
+1 w
Dadgarnia
Mar 12, 2020
zuat.e
2 minutes ago
nice geo
Melid   1
N 18 minutes ago by Melid
Source: 2025 Japan Junior MO preliminary P9
Let ABCD be a cyclic quadrilateral, which is AB=7 and BC=6. Let E be a point on segment CD so that BE=9. Line BE and AD intersect at F. Suppose that A, D, and F lie in order. If AF=11 and DF:DE=7:6, find the length of segment CD.
1 reply
1 viewing
Melid
23 minutes ago
Melid
18 minutes ago
IMO ShortList 1998, combinatorics theory problem 1
orl   44
N 31 minutes ago by YaoAOPS
Source: IMO ShortList 1998, combinatorics theory problem 1
A rectangular array of numbers is given. In each row and each column, the sum of all numbers is an integer. Prove that each nonintegral number $x$ in the array can be changed into either $\lceil x\rceil $ or $\lfloor x\rfloor $ so that the row-sums and column-sums remain unchanged. (Note that $\lceil x\rceil $ is the least integer greater than or equal to $x$, while $\lfloor x\rfloor $ is the greatest integer less than or equal to $x$.)
44 replies
orl
Oct 22, 2004
YaoAOPS
31 minutes ago
A game optimization on a graph
Assassino9931   2
N 32 minutes ago by dgrozev
Source: Bulgaria National Olympiad 2025, Day 2, Problem 6
Let \( X_0, X_1, \dots, X_{n-1} \) be \( n \geq 2 \) given points in the plane, and let \( r > 0 \) be a real number. Alice and Bob play the following game. Firstly, Alice constructs a connected graph with vertices at the points \( X_0, X_1, \dots, X_{n-1} \), i.e., she connects some of the points with edges so that from any point you can reach any other point by moving along the edges.Then, Alice assigns to each vertex \( X_i \) a non-negative real number \( r_i \), for \( i = 0, 1, \dots, n-1 \), such that $\sum_{i=0}^{n-1} r_i = 1$. Bob then selects a sequence of distinct vertices \( X_{i_0} = X_0, X_{i_1}, \dots, X_{i_k} \) such that \( X_{i_j} \) and \( X_{i_{j+1}} \) are connected by an edge for every \( j = 0, 1, \dots, k-1 \). (Note that the length $k \geq 0$ is not fixed and the first selected vertex always has to be $X_0$.) Bob wins if
\[
  \frac{1}{k+1} \sum_{j=0}^{k} r_{i_j} \geq r;
  \]otherwise, Alice wins. Depending on \( n \), determine the largest possible value of \( r \) for which Bobby has a winning strategy.
2 replies
Assassino9931
Apr 8, 2025
dgrozev
32 minutes ago
Composite sum
rohitsingh0812   39
N 40 minutes ago by YaoAOPS
Source: INDIA IMOTC-2006 TST1 PROBLEM-2; IMO Shortlist 2005 problem N3
Let $ a$, $ b$, $ c$, $ d$, $ e$, $ f$ be positive integers and let $ S = a+b+c+d+e+f$.
Suppose that the number $ S$ divides $ abc+def$ and $ ab+bc+ca-de-ef-df$. Prove that $ S$ is composite.
39 replies
rohitsingh0812
Jun 3, 2006
YaoAOPS
40 minutes ago
Problem 1
SpectralS   145
N 42 minutes ago by IndexLibrorumProhibitorum
Given triangle $ABC$ the point $J$ is the centre of the excircle opposite the vertex $A.$ This excircle is tangent to the side $BC$ at $M$, and to the lines $AB$ and $AC$ at $K$ and $L$, respectively. The lines $LM$ and $BJ$ meet at $F$, and the lines $KM$ and $CJ$ meet at $G.$ Let $S$ be the point of intersection of the lines $AF$ and $BC$, and let $T$ be the point of intersection of the lines $AG$ and $BC.$ Prove that $M$ is the midpoint of $ST.$

(The excircle of $ABC$ opposite the vertex $A$ is the circle that is tangent to the line segment $BC$, to the ray $AB$ beyond $B$, and to the ray $AC$ beyond $C$.)

Proposed by Evangelos Psychas, Greece
145 replies
SpectralS
Jul 10, 2012
IndexLibrorumProhibitorum
42 minutes ago
Help me please
sealight2107   0
44 minutes ago
Let $m,n,p,q$ be positive reals such that $m+n+p+q+\frac{1}{mnpq} = 18$. Find the minimum and maximum value of $m,n,p,q$
0 replies
sealight2107
44 minutes ago
0 replies
Rectangular line segments in russia
egxa   2
N an hour ago by mohsen
Source: All Russian 2025 9.1
Several line segments parallel to the sides of a rectangular sheet of paper were drawn on it. These segments divided the sheet into several rectangles, inside of which there are no drawn lines. Petya wants to draw one diagonal in each of the rectangles, dividing it into two triangles, and color each triangle either black or white. Is it always possible to do this in such a way that no two triangles of the same color share a segment of their boundary?
2 replies
egxa
Apr 18, 2025
mohsen
an hour ago
(help urgent) Classic Geo Problem / Angle Chasing?
orangesyrup   4
N an hour ago by Royal_mhyasd
Source: own
In the given figure, ABC is an isosceles triangle with AB = AC and ∠BAC = 78°. Point D is chosen inside the triangle such that AD=DC. Find the measure of angle X (∠BDC).

ps: see the attachment for figure
4 replies
orangesyrup
2 hours ago
Royal_mhyasd
an hour ago
Interesting combinatoric problem on rectangles
jaydenkaka   0
an hour ago
Source: Own
Define act <Castle> as following:
For rectangle with dimensions i * j, doing <Castle> means to change its dimensions to (i+p) * (j+q) where p,q is a natural number smaller than 3.

Define 1*1 rectangle as "C0" rectangle, and define "Cn" ("n" is a natural number) as a rectangle that can be created with "n" <Castle>s.
Plus, there is a constraint for "Cn" rectangle. The constraint is that "Cn" rectangle's area must be bigger than n^2 and be same or smaller than (n+1)^2. (n^2 < Area =< (n+1)^2)

Let all "C20" rectangle's area's sum be A, and let all "C20" rectangles perimeter's sum be B.
What is A-B?
0 replies
jaydenkaka
an hour ago
0 replies
Collect ...
luutrongphuc   2
N an hour ago by megarnie
Find all functions $f: \mathbb{R^+} \rightarrow \mathbb{R^+}$ such that:
$$f\left(f(xy)+1\right)=xf\left(x+f(y)\right)$$
2 replies
luutrongphuc
Apr 21, 2025
megarnie
an hour ago
hard problem
Cobedangiu   8
N 2 hours ago by IceyCold
Let $x,y,z>0$ and $xy+yz+zx=3$ : Prove that :
$\sum  \ \frac{x}{y+z}\ge\sum  \frac{1}{\sqrt{x+3}}$
8 replies
Cobedangiu
Apr 2, 2025
IceyCold
2 hours ago
China 2017 TSTST1 Day 2 Geometry Problem
HuangZhen   46
N Apr 6, 2025 by ihategeo_1969
Source: China 2017 TSTST1 Day 2 Problem 5
In the non-isosceles triangle $ABC$,$D$ is the midpoint of side $BC$,$E$ is the midpoint of side $CA$,$F$ is the midpoint of side $AB$.The line(different from line $BC$) that is tangent to the inscribed circle of triangle $ABC$ and passing through point $D$ intersect line $EF$ at $X$.Define $Y,Z$ similarly.Prove that $X,Y,Z$ are collinear.
46 replies
HuangZhen
Mar 7, 2017
ihategeo_1969
Apr 6, 2025
China 2017 TSTST1 Day 2 Geometry Problem
G H J
G H BBookmark kLocked kLocked NReply
Source: China 2017 TSTST1 Day 2 Problem 5
The post below has been deleted. Click to close.
This post has been deleted. Click here to see post.
Ghoshadi
925 posts
#36 • 2 Y
Y by Adventure10, Mango247
MNZ2000 wrote:
There is very shorter solution:
Consider polar with center at $I$ and ratio $r^2$. since $X$ is on the $EF$ so its on the polar of $A$. If $P$ is point such $DP$ is tangent to incircle, $X$ is also on the polar of $P$ so pole of $X$ is $AX$
lemma: $AX$ passes through tangent point of A-exircle with $BC$ (call this point $D'$ define $E',F'$ similary.
Proof is easy :D
We shold prove polar of $X,Y,Z$ are concurent. Means prove $AD',BF',CE'$ are concurrent
And its obvious

You think $EF$ is the polar of $A$ w.r.t. the incircle?? No it's not.
This post has been edited 1 time. Last edited by Ghoshadi, Apr 9, 2018, 2:08 AM
Z K Y
The post below has been deleted. Click to close.
This post has been deleted. Click here to see post.
Ghoshadi
925 posts
#37 • 2 Y
Y by Adventure10, Mango247
ABCDE wrote:
Let $XYZ$ be the triangle formed by the three tangents. By Desargues' Theorem, it suffices to prove that $DX$, $EY$, and $FZ$ concur.
Sorry but it is not always that $DX,EY,FZ$ concur.
Z K Y
The post below has been deleted. Click to close.
This post has been deleted. Click here to see post.
madmathlover
145 posts
#39 • 2 Y
Y by Adventure10, Mango247
Let $(I)$ be the incircle of $\triangle ABC$ and let $L, M, N$ be the touchpoints of $(I)$ with $BC, CA, AB$, respectively.

Now let $L'$ be the antipode of $L$ in $(I)$ and $L_1$ be the reflection of $L$ WRT $D$. Suppose, $AL \cap EF = L_2, AL_1 \cap L_3, AD \cap EF = D'$.

$\triangle AEF$ and $\triangle ACB$ are homothetic with homothety center $A$.
So, the homothety with center $A$ and ratio $\dfrac {1} {2}$ sends $L$ to $L_2$, $L_1$ to $L_3$ and $D$ to $D'$.
So, $L_2$ is the touchpoint of the incircle of $\triangle AEF$ with $EF$.

Now, $\triangle AEF$ and $\triangle DFE$ are also homothetic with homothety center $D'$ and ratio $-1$. Since , $D'L_2 = D'L_3$, so, $L_3$ is the touchpoint of the incircle (call it $(I')$) of $\triangle DEF$ with $EF$.

Now, it is well known that $A, L', L_1$ are collinear. Now let $AL_1 \cap (I) = L_4$. Then $\angle LL_4L_1 = 90^{\circ}$. So, $DL = DL_4$ and so $L_4$ is the touchpoint of the tangent $DX$ with $(I)$.

Now $\angle XL_3L_4 = \angle L_4L_1D = \angle DL_4L_1 = \angle XL_4L_3$ $\Rightarrow$ $XL_3 = XL_4$.

So, $X$ lies on the radical axis of $(I)$ and $(I')$.
Similarly, $Y$ and $Z$ also lie on the radical axis. So $X, Y, Z$ are collinear.
Z K Y
The post below has been deleted. Click to close.
This post has been deleted. Click here to see post.
math_pi_rate
1218 posts
#40 • 2 Y
Y by Adventure10, Mango247
My solution: Let $M,N$ be the $A$-intouch and $A$-extouch points in $\triangle ABC$. Also let $DX$ meet the incircle at $T$, and $M'$ be the antipode of $M$ in the incircle. From here, we get that $M'$ and $T$ lie on $AN$, and that $D$ is the center of $\odot (MTN)$. Let $AN \cap EF = S$. Then by the homothety centered at $A$ that sends $\triangle AEF$ to $\triangle ABC$, we get that $S$ is the $A$-extouch point in $\triangle AEF$. As $AEDF$ is a parallelogram, we have that $S$ is the $D$-intouch point in $\triangle DEF$.

Now, $\angle XST=\angle DNT=\angle DTN=\angle XTS \Rightarrow XS=XT \Rightarrow X$ lies on the radical axis of the incircles of $\triangle DEF$ and $\triangle ABC$. Similarly, $Y$ and $Z$ lie on the radical axis of the incircles of $\triangle DEF$ and $\triangle ABC$. Thus, $X,Y,Z$ are collinear. $\blacksquare$
This post has been edited 1 time. Last edited by math_pi_rate, Sep 16, 2018, 3:17 PM
Z K Y
The post below has been deleted. Click to close.
This post has been deleted. Click here to see post.
Ali3085
214 posts
#41 • 4 Y
Y by Muaaz.SY, duanby, Adventure10, Mango247
here's a different approach using poles and polars:
let $R,S,T$ the tangency points and $R',S',T'$ the second tangency points from $D,E,F$
and let $X'=SS' \cap TT'$ difine $Y',Z'$ similary
$polar(E)=TT'$
$polar(F)=RR' \implies polar (X)=X'R'$
so $X,Y,Z $ are collinear is equivalent to $X'R',Y'S',Z'T'$ are concurrent which is equivalent to $X'R,Y'S,Z'T$ are concurrent
$polar(EF \cap BC ) =X'R $
but $EF \cap BC =P_{\infty BC}$
thus $I \in X'R$ similary $I \in Y'S$ $I \in Z'T$
and we win :D
Z K Y
The post below has been deleted. Click to close.
This post has been deleted. Click here to see post.
Aryan-23
558 posts
#42 • 1 Y
Y by Mango247
Quite a simple problem; yet it took me quite a while to realize the solution. :wallbash:

Solution
This post has been edited 1 time. Last edited by Aryan-23, Feb 9, 2021, 9:32 PM
Reason: Minor correction
Z K Y
The post below has been deleted. Click to close.
This post has been deleted. Click here to see post.
ACGNmath
327 posts
#43
Y by
tenplusten wrote:
Easy with Barycentre.

Indeed, easy with barycentric coordinates.
[asy]
size(10cm);
pair A = dir(110);
pair B = dir(210);
pair C = dir(330);
pair I = incenter(A,B,C);
pair A1 = foot(I,B,C);
pair E = (A+C)/2;
pair F = (A+B)/2;
pair D = (B+C)/2;
path inc = incircle(A,B,C);
pair A4 = B+C-A1;
pair A3 = foot(A1,A,A4);
pair A2 = 2*I-A1;
pair X = extension(D,A3,F,E);
draw(A--B--C--A--cycle);
draw(inc);
draw(E--F);
draw(F--X);
draw(D--X);
draw(A--A4);
dot("$A$",A,dir(A));
dot("$B$",B,dir(B));
dot("$C$",C,dir(C));
dot("$D$",D,dir(D));
dot("$E$",E,dir(E));
dot("$F$",F,dir(F));
dot("$I$",I,dir(I));
dot("$X$",X,dir(X));
dot("$A_1$",A1,dir(A1));
dot("$A_2$",A2,dir(A2));
dot("$A_3$",A3,dir(A3));
dot("$A_4$",A4,dir(A4));
[/asy]

Let $A_1$ be the point of tangency of the incircle with side $BC$, and let $A_1 A_2$ be a diameter of the incircle. Let the other tangent from $D$ to the incircle meet the incircle at $A_3$. Then $\angle A_1 A_3 A_2=90^{\circ}$. The extension of $A_2A_3$ meets the side $BC$ at a point $A_4$. It is well-known that $A_4$ is the point of tangency of the $A$-excircle with the side $BC$. Thus,
$$A_4=(0:s-b:s-c)$$Note that the displacement vector $\overrightarrow{AA_4}$ is given by
$$\left(0,\frac{s-b}{a},\frac{s-c}{a}\right)-(1,0,0)=(-a:s-b:s-c)$$
Next, we use the following fact:
Let $(l:m:n)$ be a displacement vector. Then a displacement vector perpendicular to $(l:m:n)$ is given by
$$(a^2(n-m)+(c^2-b^2)l:b^2(l-n)+(a^2-c^2)m:c^2(m-l)+(b^2-a^2)n)$$In this case, a displacement vector perpendicular to $AA_4$ is given by
$$(2a(b-c)s:-ab^2-b^2(s-c)+(a^2-c^2)(s-b):c^2a+c^2(s-b)+(b^2-a^2)(s-c))$$Simplifying, this is equal to
$$(2a(b-c):a^2-2ab-(b-c)^2:-a^2+2ac+(b-c)^2)$$Also, $A_1=(0:s-c:s-b)$. Thus, we can parametrise the line $A_1 A_3$ is
$$(2a(b-c)t:s-c+t(a^2-2ab-(b-c)^2):s-b+(-a^2+2ac+(b-c)^2))$$Substituting this into line $AA_4$ given by $-(s-c)y+(s-b)z=0$, we get that
$$t=\frac{-b+c}{a^2-ab-ac-2b^2+4bc-2c^2}=\frac{2(b-c)}{a(s-a)+4(b-c)^2}$$From this,
$$A_3=(-4(b-c)^2:(a-b)^2-c^2:(c-a)^2-b^2)=(-4(b-c)^2:-4(s-b)(s-a):-4(s-c)(s-a))=\left(\frac{(b-c)^2}{s-a}:s-b:s-c\right)$$Similarly,
$$B_3=\left(s-a:\frac{(c-a)^2}{s-b}:s-c\right)$$$$C_3=\left(s-a:s-b:\frac{(a-b)^2}{s-c}\right)$$From this, we get that $DA_3$ is
$$(s-a)x+(b-c)y+(c-b)z=0$$Since $EF$ is given by $-x+y+z=0$, we obtain
$$X=(b-c:s-c_b-s)$$Similarly,
$$Y=(c-s:c-a:s-a)$$$$Z=(s-b:a-s:a-b)$$It remains to verify that
$$\begin{vmatrix} b-c & s-c & b-s \\ c-s & c-a & s-a \\ s-b & a-s & a-b \end{vmatrix}=0$$But this is true as the sum of the three rows is the zero row.

Thus, $X$, $Y$ and $Z$ are collinear.
Z K Y
The post below has been deleted. Click to close.
This post has been deleted. Click here to see post.
spartacle
538 posts
#44 • 3 Y
Y by SK_pi3145, Aryan-23, PRMOisTheHardestExam
One line, technically
Z K Y
The post below has been deleted. Click to close.
This post has been deleted. Click here to see post.
ike.chen
1162 posts
#45 • 1 Y
Y by Rounak_iitr
Let $I$ be the incenter, the incircle touch $BC, CA, AB$ at $A_1, B_1, C_1$ respectively, the aforementioned tangents from $D, E, F$ touch the incircle at $P_1, P_2, P_3$ respectively, $H_1$ be the orthocenter of $BIC$, $Q = CI \cap BH_1$, and $R = CI \cap EF$.

Claim: $BH_1$ is the polar of $R$ (wrt the incircle).

Proof. Since $BH_1 \perp CI \equiv IR$, it suffices to show $B$ lies on the polar of $R$.

By the Iran Lemma, $A_1C_1$ passes through $R$. But $A_1C_1$ is the polar of $B$, so the desired result follows by La Hire's. $\square$

Claim: $EF$ is the polar of $H_1$.

Proof. La Hire's implies $R$ lies on the polar of $H_1$. Because $H_1I \perp BC$ and $EF \parallel BC$, we know $IH_1 \perp EF$. But $R \in EF$, so $EF$ is the polar of $H_1$, as required. $\square$

If we let $H_2, H_3$ be the orthocenters of $CIA, AIB$ respectively, then analogous reasoning implies $FD$ is the polar of $H_2$ and $DE$ is the polar of $H_3$.

Claim: $H_1P_1$ is the polar of $X$.

Proof. Trivially, $P_1$ lies on the polar of $X$.

Since $X \in EF$, i.e. the polar of $H_1$, we know $H_1$ lies on the polar of $X$ by La Hire's. $\square$

Similarly, we conclude $H_2P_2, H_3P_3$ are the polars of $Y, Z$. By the Concurrent Polars Induces Collinear Poles Lemma, it suffices to show $H_1P_1, H_2P_2, H_3P_3$ concur.

Claim: $H_1, H_2, C_1, P_3$ are collinear.

Proof. By tangency, it's easy to see $C_1P_3$ is the polar of $F$. But we've previously shown that $F$ lies on the polars of $H_1$ and $H_2$, so both orthocenters lie on the polar of $F$ by La Hire's, which suffices. $\square$

Analogously, we conclude $H_2, H_3, A_1, P_1$ and $H_3, H_1, B_1, P_2$ are also sets of collinear points.

Now, observe $H_1A_1, H_2B_1, H_3C_1$ concur at $I$. Since $P_1, P_2, P_3$ all lie on $(A_1B_1C_1)$, the desired concurrency follows from the existence of the Cyclocevian Conjugate. $\blacksquare$
Attachments:
Z K Y
The post below has been deleted. Click to close.
This post has been deleted. Click here to see post.
rcorreaa
238 posts
#46
Y by
Trigbash for the win! :coolspeak:

Let the incircle $\omega$ touch $BC$ at $P$, $CA$ at $Q$ and $AB$ at $R$. Let $D_2$ be the antipode of $P$ and $D_1= \omega \cap AD_2$. Since $AD_2$ intersects $BC$ on the touching point of $A$-excircle, and $D_1P \perp D_1D_2$, we have that $DP=DD_1$, so $DD_1$ touches $\omega$. Furthermore, $D_1QD_2R$ is a harmonic quadrilateral, so $\frac{D_1Q}{D_1R}=\frac{D_2Q}{D_2R}=\frac{sin \angle IPQ}{sin \angle IPR}=\frac{sin \frac{C}{2}}{sin \frac{B}{2}} (\star)$
By Ratio Lemma, we know that $$\frac{XF}{XE}=\frac{sin \angle XDF}{sin \angle XDE} \frac{DF}{DE}=\frac{sin \angle XDF}{sin \angle XDE}\frac{b}{c}$$Since $DF \parallel AC$, $\angle XDF= \angle D_1DF= 180º- \angle D_1IQ$, so $sin \angle XDF= sin \angle D_1IQ= \frac{D_1Q}{2r}$, where $r$ is the inradius of $ABC$. Similarly, $sin \angle XDE= \frac{D_1R}{2r} \implies \frac{sin \angle XDF}{sin \angle XDE}= \frac{D_1Q}{D_1R}$, which is equals to $\frac{sin \frac{C}{2}}{sin \frac{B}{2}}$, from $(\star)$.
Therefore, $\frac{XF}{XE}=\frac{sin \frac{C}{2}}{sin \frac{B}{2}}\frac{b}{c}$. Analogously, $\frac{YE}{YD}=\frac{sin \frac{B}{2}}{sin \frac{A}{2}}\frac{a}{b}$ and $\frac{ZD}{ZF}=\frac{sin \frac{A}{2}}{sin \frac{C}{2}}\frac{c}{a}$. Multiplying everything, we have that $$\frac{XF}{XE}.\frac{YE}{YD}.\frac{ZD}{ZF}=1$$so we are done by Menelaus' Theorem on triangle $DEF$ WRT line $XYZ$.
$\blacksquare$
Z K Y
The post below has been deleted. Click to close.
This post has been deleted. Click here to see post.
InterLoop
274 posts
#47 • 1 Y
Y by GeoKing
orz
solution
a proof of the generalisation of the problem is stated in my post 2 posts underneath this one: https://artofproblemsolving.com/community/c6h1397197p30861050
Attachments:
This post has been edited 3 times. Last edited by InterLoop, Jun 4, 2024, 8:05 PM
Z K Y
The post below has been deleted. Click to close.
This post has been deleted. Click here to see post.
GeoKing
518 posts
#48
Y by
InterLoop wrote:
orz
solution
Is the following lemma known by carnot's theorem?
Lemma:- A circle meets the interior of $BC,CA,AB$ at $A_1,A_2;B_1,B_2;C_1,C_2$ respectively. If $AA_1,BB_1,CC_1$ concur then so does $AA_2,BB_2,CC_2$ concur
Z K Y
The post below has been deleted. Click to close.
This post has been deleted. Click here to see post.
InterLoop
274 posts
#49 • 1 Y
Y by GeoKing
GeoKing wrote:
Is the following lemma known by carnot's theorem?
Lemma:- A circle meets the interior of $BC,CA,AB$ at $A_1,A_2;B_1,B_2;C_1,C_2$ respectively. If $AA_1,BB_1,CC_1$ concur then so does $AA_2,BB_2,CC_2$ concur
this is a special case! Carnot's theorem states that even if any six points as defined above lie on a conic, the relation holds :D

anyways, here is a proof of a generalisation of the problem (instead of the medial triangle, consider any cevian triangle).

generalisation
This post has been edited 1 time. Last edited by InterLoop, Jun 4, 2024, 8:03 PM
Z K Y
The post below has been deleted. Click to close.
This post has been deleted. Click here to see post.
bin_sherlo
707 posts
#50 • 1 Y
Y by tiny_brain123
Change the tangency points to $D,E,F$ and midpoints to $M,N,P$. Let $D',E',F'$ be the antipodes of $D,E,F$ on the incircle. Let $AD',BE',CF'$ meet the incircle at $T_D,T_E,T_F$ for second time. Note that $MT_D,NT_E,PT_F$ are tangent to incircle by homothety. In order to use menelaus,
\[\Pi{(\frac{XN}{XP})}=\Pi{(\frac{\frac{\sin NMT_D.MN}{\sin MXN}}{\frac{\sin PMT_D.MP}{\sin MXN}})}=\Pi{(\frac{\sin MNT_D}{\sin PMT_D})}.\Pi{(\frac{MN}{MP})}=1\]We observe that $\Pi{(\frac{MN}{MP})}=1$.
\[\Pi{(\frac{\sin MNT_D}{\sin PMT_D})}=\Pi{(\frac{\sin FIT_D}{\sin EIT_D})}=\Pi{(\frac{T_DF}{T_DE})}=\Pi{(\frac{\sin FD'A}{\sin AD'E})}=\Pi{(\frac{AF.\frac{\sin \frac{B}{2}}{AD'}}{AE.\frac{\sin \frac{C}{2}}{AD'}})}=1\]As desired.$\blacksquare$
Z K Y
The post below has been deleted. Click to close.
This post has been deleted. Click here to see post.
ihategeo_1969
205 posts
#51
Y by
Replace $\triangle DEF$ with $\triangle M_AM_BM_C$.

All pole-polars in the proof will be with respect to $\omega$. Define some new points.
  • Let $\triangle DEF$ be the intouch triangle.
  • Let $D_1=2I-D$ and $D_2=\overline{AD_1} \cap \omega$. Then $\overline{M_AD_2}$ is tangent to $\omega$ is well known. Define $E_1$, $E_2$, $F_1$, $F_2$.
  • Let $H_A$ be orthocenter of $\triangle BIC$. Define $H_B$, $H_C$ similarly.
See that pole of $A$-midline is $\overline{FF_2} \cap \overline{EE_2}$ and similar.

Claim: $H_A$ is $\overline{EE_2} \cap \overline{FF_2}$.
Proof: We want to prove $H_A$ is pole of $A$-midline.

Let $U=\overline{BI} \cap \overline{EF}$ and $V=\overline{CI} \cap \overline{EF}$ for lack of better names. Then by Iran Lemma, we have $H_A=\overline{BV} \cap \overline{CU}$.

Now by countless La Hires, we want to show that $(AB) \cap \overline{DE}$ and $(AC) \cap \overline{DF}$ is the $A$-midline which is just Iran Lemma again. $\square$

Now $\overline{H_AD} \perp \overline{BC}$ and hence $I$ lies on $\overline{H_AD}$. Similarly it lies on $\overline{H_BE}$ and $\overline{H_CF}$.

We want to prove $\overline{H_AD_2}$, $\overline{H_BE_2}$, $\overline{H_CF_2}$ concurrent and now see that \begin{align*}
& \frac{F_2H_A}{F_2H_B} \cdot \frac{D_2H_B}{D_2H_C} \cdot \frac{E_2H_C}{E_2H_A} 
 =  \frac{\operatorname{Pow}(H_B,\omega)}
{\operatorname{Pow}(H_A,\omega)} \cdot \frac{F_2H_A}{F_2H_B} \cdot \frac{\operatorname{Pow}(H_C,\omega)}{\operatorname{Pow}(H_B,\omega)} \frac{D_2H_B}{D_2H_C} \cdot \frac{\operatorname{Pow}(H_A,\omega)}{\operatorname{Pow}(H_C,\omega)} \cdot \frac{E_2H_C}{E_2H_A} 
=  \frac{FH_B}{FH_A} \cdot \frac{DH_C}{DH_B} \cdot \frac{EH_C}{EH_A} \overset{\text{Ceva}}= 1
\end{align*}Now we are done by converse of Ceva. Taking the dual of this, we get $X$, $Y$, $Z$ collinear.
Z K Y
N Quick Reply
G
H
=
a